Doesn't A weaken?
If a company has its highest sales period during the last three months, then the company will rel...
ElleSat on February 11 at 10:55PM
  • November 2019 LSAT
  • SEC2
  • Q1
1
Reply
"a general conclusion" semantics
(1) If the incorrect answer choice which reads "it is cited as an instance of a general conclusio...
devon on May 26 at 09:02PM
  • November 2019 LSAT
  • SEC2
  • Q16
1
Reply
Intermediate conclusion
After getting this question wrong, I can see that the first sentence is an intermediate conclusio...
AndrewArabie on November 16, 2022
  • November 2019 LSAT
  • SEC2
  • Q16
2
Replies
I don't understand how B impacts the argument
I understand how C does not impact this argument. But as for B, if the demand increases, certainl...
AndrewArabie on November 8, 2022
  • November 2019 LSAT
  • SEC2
  • Q24
3
Replies
I ruled out D for the same reason the explanati...
I negated D to read "The entrapment of dire wolves in the tar pits *did not* most frequently occu...
AndrewArabie on November 8, 2022
  • November 2019 LSAT
  • SEC2
  • Q7
6
Replies
ANS choice E
In the answer choice E, it states that it is not correct because nutrients are not mentioned in t...
logan.cay34 on October 27, 2022
  • November 2019 LSAT
  • SEC2
  • Q18
1
Reply
What is the best way of answering this question...
I initially thought to diagram this question but quickly realized that the variables dont match a...
Eaten on October 5, 2022
  • November 2019 LSAT
  • SEC2
  • Q19
1
Reply
Explanation?
Can you explain why B is correct?
Eaten on October 5, 2022
  • November 2019 LSAT
  • SEC2
  • Q17
3
Replies
Explanation of A
I'm confused by this answer. Where in the stimulus are we talking about two groups of med student...
IkeHansen on September 21, 2022
  • November 2019 LSAT
  • SEC2
  • Q14
1
Reply
Could you please breakdown the argument and exp...
Thanks!
DylanMorris on September 21, 2022
  • November 2019 LSAT
  • SEC2
  • Q16
1
Reply
B vs. D
Can someone go over the conditionals in the stimulus and in answer choices B & D. I think I under...
Jasmin1 on August 2, 2022
  • November 2019 LSAT
  • SEC2
  • Q22
2
Replies
Diagramming the Conditional in the Stimulus
I got this answer choice correct because it was the closest to what I came up with, but I diagram...
skraps17 on July 31, 2022
  • November 2019 LSAT
  • SEC2
  • Q8
2
Replies
How do we get the answear?
Can anyone elaborate how you get the answer for this question?
Rikuto-Yamada on April 27, 2022
  • November 2019 LSAT
  • SEC2
  • Q14
1
Reply
A vs B
Could someone explain to me why A would not be the correct answer? I was stuck between the two, b...
RachP on April 27, 2022
  • November 2019 LSAT
  • SEC2
  • Q4
1
Reply
A vs C
While I understand why A is correct, I do not understand why C is incorrect. Is it because of the...
RachP on April 27, 2022
  • November 2019 LSAT
  • SEC2
  • Q19
1
Reply
C vs D
Can someone please explain the difference between these two answers?
RachP on April 27, 2022
  • November 2019 LSAT
  • SEC2
  • Q22
1
Reply
B v D
Can someone explain to me why D is the better answer over A?
RachP on April 27, 2022
  • November 2019 LSAT
  • SEC2
  • Q25
1
Reply
Paradox answered by a paradox?
How can the answer be that "it's the additional exercise" when the stimulus CLEARLY states "Res...
LAZR423 on March 30, 2022
  • November 2019 LSAT
  • SEC2
  • Q10
7
Replies
Explanation
Can some one explain how the other choices weaken the stimulus?
Jasmin1 on February 13, 2022
  • November 2019 LSAT
  • SEC2
  • Q24
3
Replies
Is this considered a method if reasoning question?
If so, do Method of Reasoning and Principle questions go hand in hand?
brittnysep on January 24, 2022
  • November 2019 LSAT
  • SEC2
  • Q25
1
Reply